Commuting Pauli Matrices: A Tricky Homework Challenge

Click For Summary
The discussion centers on a homework problem involving the product of Pauli matrices, specifically σy and σz. The key point is that the exponentials of these matrices cannot be combined simply because they do not commute, as indicated by their non-zero commutation relation. A suggested approach is to express the exponential as a series to better understand the relationship between the matrices. The exercise aims to highlight the importance of recognizing that matrix operations differ from scalar operations. Understanding this distinction is crucial for solving the problem correctly.
unscientific
Messages
1,728
Reaction score
13

Homework Statement



Express the product

where σy and σz are the other two Pauli matrices defined above.

commutatorpauli1.png



Homework Equations





The Attempt at a Solution



I'm not sure if this is a trick question, because right away both exponentials combine to give 1, where the result is simply σx

commutatorpauli2.png
 
Physics news on Phys.org
That's the whole point of the exercise, to see that the exponentials do not combine to give 1. To do such a thing, you would have to pass exp(i\alpha\sigma^z) to the other side of \sigma^x. But \left[\sigma^z,\sigma^x\right]\neq0 so that you can't simply commute them.

Hint: express the exponential as a series.
 
  • Like
Likes 1 person
kevinferreira said:
That's the whole point of the exercise, to see that the exponentials do not combine to give 1. To do such a thing, you would have to pass exp(i\alpha\sigma^z) to the other side of \sigma^x. But \left[\sigma^z,\sigma^x\right]\neq0 so that you can't simply commute them.

Hint: express the exponential as a series.

Ah, I see what you mean, as the sum is a matrix and not a number. Silly me.
 
Last edited:
Question: A clock's minute hand has length 4 and its hour hand has length 3. What is the distance between the tips at the moment when it is increasing most rapidly?(Putnam Exam Question) Answer: Making assumption that both the hands moves at constant angular velocities, the answer is ## \sqrt{7} .## But don't you think this assumption is somewhat doubtful and wrong?

Similar threads

Replies
4
Views
2K
  • · Replies 2 ·
Replies
2
Views
2K
  • · Replies 5 ·
Replies
5
Views
5K
  • · Replies 1 ·
Replies
1
Views
2K
  • · Replies 2 ·
Replies
2
Views
2K
Replies
3
Views
2K
  • · Replies 5 ·
Replies
5
Views
3K
  • · Replies 10 ·
Replies
10
Views
6K
Replies
9
Views
12K
  • · Replies 1 ·
Replies
1
Views
12K